For function f(x), which pair of limits would verify the existence of a vertical asymptote at x = –1?

Answers

Answer 1

The pair of limits that would verify the existence of a vertical asymptote at x = –1 is:

[tex]\lim_{x \rightarrow -1^-} = \infty, \lim_{x \rightarrow -1^+} = \infty[/tex]

What are the vertical asymptotes of a function f(x)?

The vertical asymptotes are the values of x which are outside the domain, which in a fraction are the zeroes of the denominator.

For a vertical asymptote at x = -1, the denominator is zero at -1, hence both lateral limits at x = -1 go to infinity, that is:

[tex]\lim_{x \rightarrow -1^-} = \infty, \lim_{x \rightarrow -1^+} = \infty[/tex]

More can be learned about vertical asymptotes at https://brainly.com/question/27949428

#SPJ1


Related Questions

Which expression is the simplest form of 2x^3 - x^2 + 3 (x^3 - 4x^2)

Answers

The simplest form of the given expression is 5x^3-5x^2.

We have given that

[tex]2x^3 - x^2 + 3 (x^3 - 4x^2)[/tex]

We have to determine the simplest form of the given expression.

What is the distributive property?

The distributive property of binary operations generalizes the distributive law, which asserts that equality is always true in algebra. elementary.

Use the distributive property we get,

[tex]2x^3 - x^2 + 3 (x^3 - 4x^2)\\=2x^3 - x^2 +3x^3-12x^2\\[/tex]

Add like terms we get,

Therefore we get,

[tex]=5x^3-5x^2[/tex]

Therefore the simplest form of the given expression is 5x^3-5x^2.

To learn more about the  expression visit:

https://brainly.com/question/723406

#SPJ1

$Need help with this pls (quickest answer gets brainliest)

Answers

Answer:

-1/2

Step-by-step explanation:

IK

Ahman has a lawn care business. he charges $25 per lawn to mow the grass. if his monthly expenses are $100, how many lawns must he mow in order to make a profit of at least $250 per month?

Answers

Answer:

14

Step-by-step explanation:

you can write an equation to represent the situation:

let x represent the number of lawns he has to mow

then, the monthly profit would be 25x - 100.

in order to make $250, this equation has to be equal to 250:

[tex]25x-100=250[/tex]

now, solve this:

add 100 to both sides

[tex]25x = 350[/tex]

divide both sides by 25

[tex]x = 14[/tex]

he must mow 14 lawns

Round 508.0219 to the nearest hundredth

Answers

Answer:

508.02

HOPE THIS HELPS

Todo <3

Step-by-step explanation:

CAN SOMEONE HELP ME PLS?????

Answers

You are correct, P(red) = P(green) = 0.25.

Find the slope of every line that is parallel to line on the graph (0,-3) (5,-4)

Answers

The slope of the line parallel to the line on the graph (0,-3) (5,-4) is - 1 / 5

How to find the slope of a parallel lines?

Parallel lines have the same slope.

Therefore,

slope = m = y₂ - y₁ / x₂ - x₁

Therefore,

x₁ = 0

x₂ = 5

y₁ = -3

y₂ = -4

Therefore,

slope = -4 - (-3)  / 5 - 0

slope = -4 + 3 / 5

slope = - 1 / 5

learn more on slope here: https://brainly.com/question/8978282

#SPJ1

A passenger jet plane cruises at 550 knots. It enters a jet stream with
a tailwind speed of 120 knots. If the speed of sound is 661 knots,
will the jet travel faster or slower than sound during its journey?

Answers

Considering the direction of the wind, it is found that the jet travels faster than the sound during it's journey.

What is the ground speed?

The jet's speed, considering that there is a tailwind, is given by:

J = Plane speed + Wind speed.

In this problem, we have that the speeds are given as follows:

Plane: 550 knots.Wind: 120 knots.

Hence the jet's speed is given by:

J = 550 + 120 = 670 knots.

670 knots > 661 knots, hence the jet travels faster than the sound during it's journey.

A similar problem, involving plane and wind's speed, is given at brainly.com/question/25547425

#SPJ1

sum of numbers 975,983,923,913 and 985 rounded upto hundredth place is

Answers

4,779 if you are just adding to find the sun of the numbers

Answer:

4800

Step-by-step explanation:

If a question is asking for the "sum" of numbers, this just means we have to add them altogether.

975+983+923+913+985 = 4779

To find the hundredth place, we use our place value. 4 is in the thousands column, and 7 is in the hundreds. Once we find the hundreds, we need the number to the right of it (7).

7 > 5 so we round up. 7 becomes 8.

4800.

pls help
Enter the equation for the graph.

Answers

Answer:

[tex]y=[1]cos([\frac{2\pi }{3}]x)[/tex]

Step-by-step explanation:

Looking at the graph, we can see the domain to be from (0 , 2π).

Now we have to find one period that corresponds to cos(x).

The half-period of cos(x) for this graph appears to be pi/3 and adding another pi/3 gets us 2pi/3 to be our cosine period.

b = 2pi/3

a is the same range as cos(x). Range: (0,0)

y = [a] * cos ([b]*x)

y = [1] * cos([2pi/3]x)

Select the statement below that correctly describes the relationship between the number of workers and the hours that they work.

Answers

Less workers = more hours

OR the answer might be worded as

More workers = less hours


If the triangles above are reflections of each other, then BC = to

Answers

Answer:

DF

Step-by-step explanation:

Paint a picture in your head reflecting the triangle. Just hold A and flip it so A = E , D = B

The temperature fell at a rate of 0.65 °C/h. The temperature was recorded at 37 °C
at 6 p.m. Which function can be used to represent this situation?
Of(x) = 37 -0.65x
Of(x) = 0.65x - 37
Of(x) = 37x + 0.65
Of(x) = 0.65x+37

Answers

The linear function that can be used to represent the temperature in x hours after 6 pm is given by:

f(x) = 37 - 0.65x.

What is a linear function?

A linear function is modeled by:

y = mx + b

In which:

m is the slope, which is the rate of change, that is, by how much y changes when x changes by 1.b is the y-intercept, which is the value of y when x = 0, and can also be interpreted as the initial value of the function.

In this problem:

The y-intercept is the initial temperature of 37ºC.The slope is the rate of change of -0.65ºC/h.

Hence the function is:

f(x) = 37 - 0.65x.

More can be learned about linear functions at https://brainly.com/question/24808124

#SPJ1


[tex]\{ \frac { ( \sqrt { 3 } ) \times 3 ^ { - 2 } } { ( \sqrt { 5 } ) ^ { 2 } } \} ^ { \frac { 1 } { 2 } }[/tex]solve this equation

Answers

Answer:

Step-by-step explanation:

Exponent law:

    [tex]\sf \bf a^m * a^n = a^{m+n}\\\\ (a^m)^n = a^{m*n}[/tex]

    [tex]\sf a^{-m}=\dfrac{1}{a^m}[/tex]

       First convert radical form to exponent form and then apply exponent law.

 [tex]\sf \sqrt{3}=3^{\frac{1}{2}}\\\\\sqrt{5}=5^{\frac{1}{2}}[/tex]

[tex]\sf \left(\dfrac{(\sqrt{3}*3^{-2}}{(\sqrt{5})^2}\right)^{\frac{1}{2}}= \left(\dfrac{3^{\frac{1}{2}}*3^{-2}}{(5^{\frac{1}{2}})^2} \right )^{\frac{1}{2}}[/tex]

                      [tex]= \left(\dfrac{3^{\frac{1}{2}-2}}{5^{\frac{1}{2}*2}}\right)^{\frac{1}{2}}\\\\=\left(\dfrac{3^{\frac{1-4}{2}}}{5}\right)^{\frac{1}{2}}\\\\=\left(\dfrac{3^{\frac{-3}{2}}}{5}\right)^{\frac{1}{2}}\\\\=\dfrac{3^{\frac{-3}{2}*{\frac{1}{2}}}}{5^{\frac{1}{2}}}\\\\ =\dfrac{3^{{\frac{-3}{4}}}}{5^{\frac{1}{2}}}[/tex]

The lowest point in the United States is the Death Valley in California. Its altitude is 282 feet below sea level. Identify the integer representing the lowest point (Death Valley).

Answers

Answer:

-282

Step-by-step explanation:

Any altitude below sea level would have a negative sign in front of it.

PLEASE HELP 15 POINTS

Answers

Answer:

B

Step-by-step explanation:

Used elimination method.

first plugged in (0,0)which remove A,D

than saw the line is not solid so chose B

8. Complete the table below by drawing an example of each shape.

Answers

Answer:

Please see attachment.

The first one is a quadrilateral that is not a parallelogram.

The second one is a triangle with a right angle.

Step-by-step explanation:

Hope this helps!

If not, I am sorry.

answer?.............

Answers

Answer:

-21

Step-by-step explanation:

My guess would be -21 Im not sure but that is my guess because x=5 the normal answer would be negative 25 so my guess would negative 21

4
Find the measure of x.
X
12°
X = = [?]
Round to the nearest hundredth.

Answers

The measure of x will be 18.81 units.

What are trigonometric identities?

Trigonometric identities are the functions that include trigonometric functions such as sine, cosine, tangents, secant, and, cot.

The given information are;

Perpendicular = 4

Base = x

Then, Tan ∅ = Perpendicular /Base

Tan 12 = 4/x

x = 4 /0.212

x = 18.818

Hence, the measure of x will be 18.81 units.

Learn more about trigonometric;

https://brainly.com/question/21286835

#SPJ1

Andrea has 3 tiles. One is a regular octagon, one is a regular pentagon and one is a regular hexagon. Andrea thinks the 3 tiles will fit together perfectly, as shown in the diagram. Show calculations to prove that she is wrong.

Answers

From calculations, we can say that the given tiles will not fit together perfectly.

How to find the sum of interior angles of a Polygon?

If the tiles join perfectly at a point, sum of all angles around the joining point should be 360°.

Expression for the measure of the interior angle of a polygon,

Interior angle of a polygon = [(n - 2) * 180]/n

Interior angle of a pentagon = [(5 - 2) * 180]/5 = 108°

Interior angle of a hexagon = [(6 - 2) * 180]/6 = 120°

Interior angle of an octagon = [(8 - 2) * 180]/8 = 135°

To prove that the given tiles fit together perfectly → Sum of all the angles around the common point should be 360°

Sum of all interior angles = 108° + 120° + 135° = 363°

Therefore, given tiles will not fit together perfectly.

Read more about Interior angles of a Polygon at; https://brainly.com/question/224658

#SPJ1

Find all solutions of the equation in the interval .
Write your answer in radians in terms of .
If there is more than one solution, separate them with commas.

Answers

The solutions to the trigonometric equation in the desired interval are given as follows:

[tex]\theta = \frac{\pi}{3}, \theta = \frac{5\pi}{3}[/tex]

What is the solution to the trigonometric equation?

The trigonometric equation is given by:

[tex]\sqrt{3}\cot{\theta} - 1 = 0[/tex]

Solving it similarly to an equation, we have that:

[tex]\sqrt{3}\cot{\theta} = 1[/tex]

[tex]\cot{\theta} = \frac{1}{\sqrt{3}}[/tex]

Since [tex]\cot{\theta} = \frac{1}{\tan{\theta}}[/tex], we have that the equation is equivalent to:

[tex]\tan{\theta} = \sqrt{3}[/tex]

The tangent is positive in the first and in the fourth quadrant. In the first quadrant, the angle [tex]\theta[/tex] with [tex]\tan{\theta} = \sqrt{3}[/tex] is:

[tex]\theta = \frac{\pi}{3}[/tex]

In the fourth quadrant, the equivalent angle is:

[tex]\theta = 2\pi - \frac{\pi}{3} = \frac{5\pi}{3}[/tex]

More can be learned about trigonometric equations at https://brainly.com/question/24680641

#SPJ1

8 1/2 - 3/8=?
pls answer fast ​

Answers

Answer:

65/8

Step-by-step explanation:

1) Turn all numbers into improper fractions: 17/2 - 3/8 = ?

2) Make all denominators the same number by finding the least common factor, which is 8. Multiply the denominator in 17/2, which is 2, by 4 to match the other denominator. And then multiply the numerator (17) by 4 as well so that the fraction still has the same value: 68/8 - 3/8 = 65/8

3) Can not simplify since there are no common factors between 65 and 8.

Need Help Fast!!!!!! The graph of the piecewise function f(x) is shown. f(x) What is the range of f(x)?

Answers

Answer:

The second option

Step-by-step explanation:

If you look at the graph, it appears that from negative infinity to 0, the line is just constant, so the range of that would simply be the constant value or in this case 4. from 0 to infinity it appears the line is decreasing at a constant rate and should go towards negative infinity as x goes towards infinity. So the range would be -infinity < f(x) <= 4

Which expression is equivalent to startfraction (4 p superscript negative 4 baseline q) superscript negative 2 baseline over 10 p q superscript negative 3 baseline endfraction? assume p not-equals 0, q not-equals 0.

Answers

The given expression is equivalent to [tex]\frac{p^{7}q}{160}[/tex]

What are indices?

An index is a small number that tells us how many times a term has been multiplied by itself.

The plural of index is indices.

Below is an example of a term written in index form :[tex]4^{3}[/tex]

4 is the base and 3 is the index.

We can read this as ‘4 to the power 3’

Another way of expressing [tex]4^{3}[/tex] is

4 x 4 x 4 = 64

Indices can be positive or negative numbers.

Given expression can be written as [tex]\frac{({4p^{-4}q})^{-2}}{10pq^{-3}}[/tex]

Now to simplify the given fractional expression :[tex]\frac{({4p^{-4}q})^{-2}}{10pq^{-3}}[/tex]

=   [tex]\frac{4^{-2}p^{8}q^{-2}}{10pq^{-3}}[/tex]                    By using the property of exponents is given by:

                                    [tex](a^{m})^{n}=a^{m n}[/tex]

=[tex]\frac{p^{7}q}{10 .16}[/tex]                              By using the property of exponents given by

                                       [tex]a^{m}a^{n}=a^{m + n}[/tex]  and  [tex]a^{-m}= \frac{1}{a^{m}}[/tex]

= [tex]\frac{p^{7}q}{160}[/tex]

Learn about indices here :

https://brainly.com/question/27327380

#SPJ4

its math help me out?

Answers

Its the first one

--------------------

Answer:

  $24 = $0.40(60)

Step-by-step explanation:

Match the input value and its location in the equation.

__

  $24 = $0.40(60)

_____

Additional comment

When input is liters and output is dollars, the constant of proportionality must have units of "dollars per liter." The dollar sign of these units is not shown in the left panel, but is shown on the answer choices. If you understand units conversion, this should not be a mystery. (The mystery is why the curriculum materials are inconsistent.)

what does 5r-r-9=15 look like after combining like terms ?
and whats the answer for r?

Answers

The value of r from the expression is 6

Simplifying expression

Given the expression below;

5r-r-9=15

Add 9 to both sides

5r - r - 9 + 9 = 15 + 9

4r = 15 + 9

4r = 24

After combining the like terms the expression will be 4r = 24

4r/4 = 24/4

r = 6

Hence the value of r from the expression is 6

Learn more on expression here: https://brainly.com/question/723406

#SPJ1

Each of 36 students at a school play bought either a cup of orange juice or a sandwich. A cup of orange juice costs $1 and a sandwich costs $3. The total amount collected was $76. How many students bought orange juice, and how many bought a sandwich?

Let represent the number of students who bought a cup of orange juice and represent the number of students who bought a sandwich. Then the problem can be represented by this system of equations:

+ 3 = 76
+ = 36

Answer the questions to solve the problem.

1. Explain what you should do with the two equations to eliminate one of the variables.

Answers

In order to eliminate one of the variables, subtract one of the equation from the other equation.

How to eliminate one of the variables?

Given these equations:

o + 3s = 76 equation 1

o + s = 36 equation 2

Where:

o = number of orange juice bought

s = number of sandwiches bought

In order to eliminate one of the variables, subtract equation 2 from equation 1. The result is :

2s = 40

s = 20

To learn more about simultaneous equations, please check: https://brainly.com/question/25875552

#SPJ1

. If we start with one bacterium and it doubles in
number every 15 minutes, HOW LONG WILL IT
TAKE for one bacterium to become one million??

Answers

Answer:

298.97 minutes

Step-by-step explanation:

2^x = 1 000 000      where 'x' is the number of 15 minute periods

x = 19.931      15 minute periods = 298.97 minutes

Mr. Lopez fruit salad recipe requires 3/4 of a cup of fresh peaches for 1 serving. He uses 9 cups of fresh peaches to prepare the salad. How many servings of the fruit salad did he prepare?

Answers

Answer:

6.75 servings

Step-by-step explanation:

[tex] \frac{3}{4} \times 9 = \frac{27}{4} = 6 \frac{3}{4} = 6.75[/tex]

Answer:

9 servings

Step-by-step explanation:

feel free to ask where you don't understand.

Gene has a gasoline budget of $300 per month. He uses an average of $6 of gasoline each day he drives. Which of the following equations represents how much money is left in his gasoline budget after x days of driving?

Answers

Answer:

300-6x

Step-by-step explanation:

Trust me

The nature of a graph, which has an even degree and a positive leading coefficient will be

Answers

The nature of a graph, which has an even degree and a positive leading coefficient will be up left, up right position

What is the nature of the graph of a quadratic equation?

The nature of the graphical representation of a quadratic equation with an even degree and a positive leading coefficient will give a parabola curve.

Given that we have a function f(x) = an even degree and a positive leading coefficient. i.e.

y = f(x) = 2x²+ 4

The domain of this function varies from -∞ < x < ∞ and the parabolic curve will be positioned on the upward left and upward right x-axis.

Learn more about the graph of a quadratic equation here:

https://brainly.com/question/9643976

#SPJ1

Other Questions
The population of discouraged workers causes the: Group of answer choices actual unemployment rate to overstate the true level of unemployment. actual unemployment rate to increase when the workers are not included in the unemployment calculation. actual unemployment rate to understate the true level of unemployment. natural rate of unemployment to increase. The elements in a defamation case are: a. defamatory statement; falsity; communication; and injury. b. a contract; knowledge of the contract; improper inducement; injury. c. false or misleading fact statements; statements in commercial advertising; likelihood of harm. d. duty; breach of duty; proximate causation; and damages. The stock of Company A lost 2% today to $73.50. What was the opening price of the stock in the beginning of the day? Help me with this question please!! :) limestone which physical property contributes to its usefulness and how? ALBERTO: Tami, Ive got a problem. Can we talk for a minute?TAMI: Sure, Alberto, whats up?ALBERTO: Last week my sous-chef, Ricky, bought tomatoes that just werent ripe. It made me really angry. Well never keep our Michelin star unless we maintain the quality of our produce!TAMI: Wow, Alberto, that sounds like a real problem. So youre saying that Ricky bought tomatoes you couldnt serve?ALBERTO: Yes! And its not the first time...TAMI: Ill do something about this right away.What component of reflective listening is missing from this conversation? Affirming contact Clarifying the implicit Paraphrasing the expressed the map is drawn to the scale of 1:4000 and the distance in between two places on the map is 4 cm what is the actual distance between those two places Please help!!! I will give 20 points to the correct answer !! Thanks so much why does the structure of the plasma membrane make this type of transport neccesary for fluids Need to know asap thanks The sequence an = 1(3)n 1 is graphed below: coordinate plane showing the points 1, 1; 2, 3; and 3, 9 Find the average rate of change between n = 1 and n = 3. (6 points) ___________ provide motility to a sperm cell, ___________ act as sensory "antennae" in many cells and ____________ Increase a cell surface area In the play about Tony and Maria, Anita tells the Jets that Chino killed Maria.TrueFalse? Where is UTC the local time? What is the verb in this sentence Use the commutative property to simplify the expression 4/3 + 3/4 + 2/3 Instructions: Rewrite the equation in Slope-Intercept Form. y2=2/5(x5) Use Table B in your Student Guide to answer the questions about ion concentrations.A solution with a pH = 13 has approximately how many moles of OH ions per liter?How many moles of H* would this same solution have per liter?(Use the decimal form of your answer.)A different solution with an H+ concentration of 1.0 x 10-4 would have a pH = Who was the leader of the war hawks for the southern states, advocating going to war with Britain in 1812? A.James MadisonB.John MarshallC.Henry ClayD.John C. CalhounIt is not C or A The major advocate of predestination was Martin Luther.TrueFalse